Você está na página 1de 6

Final Exam*

REAL ANALYSIS II


June 22, 2016

Submitted to

Dr. Legesse Lemecha


* Miliyon

T.

Part I

(10 points) Write your answer neatly and precisely on the space provided.
1. State:
(a) Minkowski Inequality
Theorem 1.1. If f and g are in Lp with 1 p , then so is f + g and
kf + gkp kf kp + kgkp
If 1 < p < , then the equality can hold only if there are nonnegative constants & such that
f = g.
(b) H
older Inequality
Theorem 1.2. If p and q are nonnegative extended real numbers such that
f Lp and g Lq , then f g L1 and
Z
|f g| kf kp kgkq

1
p

1
q

= 1 and if

Equality holds iff for some , constants not both zero we have |f |p = |g|q a.e.
(c) Radon-Nikodym Theorem
Theorem 1.3. Let (X, M, ) be a finite measure space and let be a measure defined on M
such that  . Then there exists a non negative measurable function f defined on X such that
Z
(E) =
f d for all E M.
E

The function f is unique in the sense that if g is any nonnegative measurable function on X that
also has this property, then g = f a.e. [].
(d) Fubini Theorem
Theorem 1.4. Let f : X Y R be integrable function. Then the iterated integrals exist
and
Z
Z Z
Z Z
f d( ) =
f dd =
f dd
(e) Tonelli Theorem
Theorem 1.5. Let (X, A, ) and (Y, B, ) be a finite measure
R R space andRlet
R f : X Y R
be a measurable function. If one of the iterated integrals
|f |dd or
|f |dd exists,
then the function f is integrable and hence the other iterated integrals exists and
Z Z
Z Z
Z Z
f d( ) =
f dd =
f dd

Part II

(30 points) Workout Problems: Show all the necessary steps neatly and precisely
on the space provided.
1. Let 1 p find the value the parameter for which
Z
1
lim
f = 0, for all f Lp ([0, 1])
0+ 0

Solution. Let 1 < p < and

1
p

1
q

= 1. et g = [0,] . Then
Z

q[0,] d

kgkq =

1/q

= 1/q

Using Holders inequality,


Z

[0,] |f |d 1/q kf k.

Then

lim

0+

f d = 0.
[0,]

If
lim+

1/q kf kp
= 0 lim+ 1/q kf kp = 0

0
lim+ 1/q = 0
0

1
1
1
>0< =1
q
q
p

If p = 1,
Z
lim+

[0,]

f d = 0 lim+
0

k
=0<0

If p = ,
lim+

Z
[0,]

f d = 0 lim+
0

1
kf k

2. Assume that f L3 ([0, 1]). Show that


Solution. Let g(x) =

1
x

R1
0

Z
[0,]

f (x)
dx
x

d kf k lim+
0

1
1
= lim+ 1 = 0 < 1.

< .

and p = 3. Then the conjugate exponent is


1
1
2
3
1
=1 =1 = q = .
q
p
3
3
2

Now
Z

|g(x)|

3/2

Z 1
dx =
0

3/2

Z
dx =

gL

[0,1]

3/4

1

1/4

=4<
0

[0, 1].

Z
Z

f (x)
dx =
f (x)g(x)dx
|f ||g(x)|dx
x
[0,1]
[0,1]
Z
1/3 Z
2/3
3
3/2

|f | dx
|g(x)| dx
[0,1]

[0,1]

kf k3 kgk3/2 <
Therefore

dx = 4x

0
3/2

By Holders inequality
Z


Z
[0,1]

f (x)
dx < .
x

3. Let a > 0. Prove that

Z
n

n2 xen x
dx
1 + x2

lim

exists and find its value. Justify your argument.


Solution.
2

n2 xen x
dx = lim
k
1 + x2

Z
a

nk

= lim

n2 xen x
dx
1 + x2

na

xex
2 dx
1 + nx2
2

Z
= lim

[na,nk]
[0,)

xex
2 dx
1 + nx2

By MCT
2

Z
=

(na,)
[0,]

xex
2 dx
1 + nx2


xex2
x2

= g(x)
1 + x2 xe
n2
n2

Z
2
2
1
1
=
xex dx = ex =
2
2
0
0


x2

(na,) xe 2

1+ x
Z

xex

[0,)

g L1
Thus by LDCT
Z

lim

n2 xen x
dx =
1 + x2

Therefore

Z
lim

Z
lim [na,)

[0,) n
2

xex
2 dx = 0
1 + nx2

n2 xen x
dx = 0.
1 + x2

4. Let (X, M, ) be a measure space for which 0 < (X) < , and suppose that f L (X, M, ).
i. Show that f Lp (X, M, ).
ii. Prove that
lim kf kp = kf k

Solution. i.

R
X

|f |p d supxX |f |p

R
X

d kf kp (X) <
f Lp (X, M, )

ii. (

R
X

|f |p d)1/p (kf kp )1/p ((X))1/p


Z
lim

1/p
|f |p d
lim (kf kp )1/p lim ((X))1/p
p

Z
lim

1/p
|f | d
kf k
p

lim sup kf kp kf k

(1)

Assume that M = kf k > 0. Given 0 < < M , there is a set A M such that (A) > 0 and
|f | > M on all of A M. Therefore, for all p > 0
Z
|f |p d (M )p (A)
X

Z

1/p

|f | d

(M )(A)1/p

Z

1/p

|f | d

lim inf

lim (M ) lim (A)1/p


p

lim inf kf kp M = kf k
p

Since is arbitrary, we have


lim inf kf kp kf k

(2)

Combining (1) and (2) we have


lim kf kp = kf k

5. Let 1 , 2 and be measures on a measure space (X, M, ). Show that if 1  and 2  then
(1 + 2 )  .
Solution. Let E be a measurable set in M such that (E) = 0. Since 1  and 2  ,
1 (E) = 0 = 2 (E). Thus, (1 + 2 )(E) = 1 (E) + 2 (E) = 0. Therefore, (1 + 2 )  .
6. Compute
Z

ey

sin2 (y)
dy
y

Hint: Take f (x, y) = sin(2xy)ey and apply Fubinis theorem.


Solution.
ey

Z
0

Let I(x) =

R
0

sin(2xy)dx = ey ( cos(2xy))|1x=0


1 cos(2y)
ey sin2 (y)
= ey
=
2y
y

sin(2xy)ey dy. Applying integration by parts twice we have


Z

2
I(x) = 2x 4x
sin(2xy) ey dy
0

I(x) = 2x 4x2 I(x) I(x)[1 + 4x2 ] = 2x


I(x) =
Thus

Z 1 Z
0


Z
sin(2xy)ey dy dx =

2x
1 + x2

1

2x
1
log 5
2
dx
=
log(1
+
4x
)
1= 4
2
1 + 4x
4
0

Fubinis Theorem applicable because



Z Z 1
Z
sin(2xy)ey dx dy
0

Z 1
0

Therefore
Z
0

ey


Z
ey dx dy =

sin2 (y)
log 5
dy =
y
4

ey dy = 1 < .

Você também pode gostar